[GiNaC-list] question about lorentz_g contract with sy_anti tensor

Vladimir V. Kisil kisilv at maths.leeds.ac.uk
Sun May 26 13:02:25 CEST 2019


>>>>> On Sun, 26 May 2019 10:23:56 +0000, Feng Feng <F.Feng at outlook.com> said:

    FF> Thanks very much for your so quick reply.  Yes, my mistake, I
    FF> actually mean A^{abc}_{c} = A^{abcd} * g_{dc}


    FF> So it goes back to my original post, it seems GiNaC will not
    FF> produce a 0 result.

    Raising-lowering indices can break symmetry of tensors, see the
  second answer here:

https://physics.stackexchange.com/questions/315970/is-a-tensor-which-is-symmetric-in-two-indices-still-symmetric-after-raising-lowe
-- 
Vladimir V. Kisil                 http://www.maths.leeds.ac.uk/~kisilv/
  Book:     Geometry of Mobius Transformations     http://goo.gl/EaG2Vu
  Software: Geometry of cycles          http://moebinv.sourceforge.net/


More information about the GiNaC-list mailing list